Probability of chossing two points from a segment of length L so that one is 2L/3 greater than another.Probability of one normdist being greater than anotherProbability that distance between $X$ and $Y$ is $>$ $L/3$How to set the limits of integration to find the probability of the distance between two pointsTwo points are selected on a straight line of length 'a' units at randomProbability that for two random points in unit ball, one is closer to the center than to the other pointTwo points are randomly selected on a line of length $1$Probability of one of the 3 independent uniform random variable being greater than rest of two.How to find the probability that the average of X and Y is less than 15?Choosing 2 Points on A lineWhat is the probability two independent exponentially distributed random variables being greater than one another?

How much of data wrangling is a data scientist's job?

How can saying a song's name be a copyright violation?

What mechanic is there to disable a threat instead of killing it?

What to put in ESTA if staying in US for a few days before going on to Canada

Why is it a bad idea to hire a hitman to eliminate most corrupt politicians?

Intersection of two sorted vectors in C++

Took a trip to a parallel universe, need help deciphering

Could gravitational lensing be used to protect a spaceship from a laser?

In a spin, are both wings stalled?

Would Slavery Reparations be considered Bills of Attainder and hence Illegal?

Why "Having chlorophyll without photosynthesis is actually very dangerous" and "like living with a bomb"?

Is "remove commented out code" correct English?

Where does SFDX store details about scratch orgs?

What do you call someone who asks many questions?

Doing something right before you need it - expression for this?

Why is the 'in' operator throwing an error with a string literal instead of logging false?

Is it canonical bit space?

Can a rocket refuel on Mars from water?

Brothers & sisters

If human space travel is limited by the G force vulnerability, is there a way to counter G forces?

Western buddy movie with a supernatural twist where a woman turns into an eagle at the end

Is it legal for company to use my work email to pretend I still work there?

Blender 2.8 I can't see vertices, edges or faces in edit mode

How could indestructible materials be used in power generation?



Probability of chossing two points from a segment of length L so that one is 2L/3 greater than another.


Probability of one normdist being greater than anotherProbability that distance between $X$ and $Y$ is $>$ $L/3$How to set the limits of integration to find the probability of the distance between two pointsTwo points are selected on a straight line of length 'a' units at randomProbability that for two random points in unit ball, one is closer to the center than to the other pointTwo points are randomly selected on a line of length $1$Probability of one of the 3 independent uniform random variable being greater than rest of two.How to find the probability that the average of X and Y is less than 15?Choosing 2 Points on A lineWhat is the probability two independent exponentially distributed random variables being greater than one another?













2












$begingroup$


Two points are selected randomly on a line of length $L$ so as to be on the opposite sides of the midpoint of the line. In other words, two points X and Y are independent random variables such that X is uniformly distributed over $(0,L/2)$ and Y is so over $(L/2,0)$. Find the probability that the distance between these two points is greater than $2L/3$.



Here's what I have done.



enter image description here



Can I get some help to finish this.










share|cite|improve this question











$endgroup$







  • 1




    $begingroup$
    Not help finishing but a suggestion on how to start. You can assume $L=2$. Then find the area of the part of the unit square that satisfies your condition. (This may in fact be what you've done, with the picture making the calculations clearer,.)
    $endgroup$
    – Ethan Bolker
    Mar 21 at 14:31










  • $begingroup$
    Ah. Quite good idea. Thanks.
    $endgroup$
    – ChakSayantan
    Mar 21 at 14:47










  • $begingroup$
    Let $L=2$ and define $A=1-X$, $B=Y-1$, then $Y-X=A+B$. $A$ and $B$ are then i.i.d. over $(0,1)$ and you are looking for $P(A+Bgefrac43)$
    $endgroup$
    – Daniel Mathias
    Mar 21 at 14:52















2












$begingroup$


Two points are selected randomly on a line of length $L$ so as to be on the opposite sides of the midpoint of the line. In other words, two points X and Y are independent random variables such that X is uniformly distributed over $(0,L/2)$ and Y is so over $(L/2,0)$. Find the probability that the distance between these two points is greater than $2L/3$.



Here's what I have done.



enter image description here



Can I get some help to finish this.










share|cite|improve this question











$endgroup$







  • 1




    $begingroup$
    Not help finishing but a suggestion on how to start. You can assume $L=2$. Then find the area of the part of the unit square that satisfies your condition. (This may in fact be what you've done, with the picture making the calculations clearer,.)
    $endgroup$
    – Ethan Bolker
    Mar 21 at 14:31










  • $begingroup$
    Ah. Quite good idea. Thanks.
    $endgroup$
    – ChakSayantan
    Mar 21 at 14:47










  • $begingroup$
    Let $L=2$ and define $A=1-X$, $B=Y-1$, then $Y-X=A+B$. $A$ and $B$ are then i.i.d. over $(0,1)$ and you are looking for $P(A+Bgefrac43)$
    $endgroup$
    – Daniel Mathias
    Mar 21 at 14:52













2












2








2





$begingroup$


Two points are selected randomly on a line of length $L$ so as to be on the opposite sides of the midpoint of the line. In other words, two points X and Y are independent random variables such that X is uniformly distributed over $(0,L/2)$ and Y is so over $(L/2,0)$. Find the probability that the distance between these two points is greater than $2L/3$.



Here's what I have done.



enter image description here



Can I get some help to finish this.










share|cite|improve this question











$endgroup$




Two points are selected randomly on a line of length $L$ so as to be on the opposite sides of the midpoint of the line. In other words, two points X and Y are independent random variables such that X is uniformly distributed over $(0,L/2)$ and Y is so over $(L/2,0)$. Find the probability that the distance between these two points is greater than $2L/3$.



Here's what I have done.



enter image description here



Can I get some help to finish this.







probability probability-theory conditional-probability






share|cite|improve this question















share|cite|improve this question













share|cite|improve this question




share|cite|improve this question








edited Mar 21 at 14:32









Ethan Bolker

45.6k553120




45.6k553120










asked Mar 21 at 14:24









ChakSayantanChakSayantan

492511




492511







  • 1




    $begingroup$
    Not help finishing but a suggestion on how to start. You can assume $L=2$. Then find the area of the part of the unit square that satisfies your condition. (This may in fact be what you've done, with the picture making the calculations clearer,.)
    $endgroup$
    – Ethan Bolker
    Mar 21 at 14:31










  • $begingroup$
    Ah. Quite good idea. Thanks.
    $endgroup$
    – ChakSayantan
    Mar 21 at 14:47










  • $begingroup$
    Let $L=2$ and define $A=1-X$, $B=Y-1$, then $Y-X=A+B$. $A$ and $B$ are then i.i.d. over $(0,1)$ and you are looking for $P(A+Bgefrac43)$
    $endgroup$
    – Daniel Mathias
    Mar 21 at 14:52












  • 1




    $begingroup$
    Not help finishing but a suggestion on how to start. You can assume $L=2$. Then find the area of the part of the unit square that satisfies your condition. (This may in fact be what you've done, with the picture making the calculations clearer,.)
    $endgroup$
    – Ethan Bolker
    Mar 21 at 14:31










  • $begingroup$
    Ah. Quite good idea. Thanks.
    $endgroup$
    – ChakSayantan
    Mar 21 at 14:47










  • $begingroup$
    Let $L=2$ and define $A=1-X$, $B=Y-1$, then $Y-X=A+B$. $A$ and $B$ are then i.i.d. over $(0,1)$ and you are looking for $P(A+Bgefrac43)$
    $endgroup$
    – Daniel Mathias
    Mar 21 at 14:52







1




1




$begingroup$
Not help finishing but a suggestion on how to start. You can assume $L=2$. Then find the area of the part of the unit square that satisfies your condition. (This may in fact be what you've done, with the picture making the calculations clearer,.)
$endgroup$
– Ethan Bolker
Mar 21 at 14:31




$begingroup$
Not help finishing but a suggestion on how to start. You can assume $L=2$. Then find the area of the part of the unit square that satisfies your condition. (This may in fact be what you've done, with the picture making the calculations clearer,.)
$endgroup$
– Ethan Bolker
Mar 21 at 14:31












$begingroup$
Ah. Quite good idea. Thanks.
$endgroup$
– ChakSayantan
Mar 21 at 14:47




$begingroup$
Ah. Quite good idea. Thanks.
$endgroup$
– ChakSayantan
Mar 21 at 14:47












$begingroup$
Let $L=2$ and define $A=1-X$, $B=Y-1$, then $Y-X=A+B$. $A$ and $B$ are then i.i.d. over $(0,1)$ and you are looking for $P(A+Bgefrac43)$
$endgroup$
– Daniel Mathias
Mar 21 at 14:52




$begingroup$
Let $L=2$ and define $A=1-X$, $B=Y-1$, then $Y-X=A+B$. $A$ and $B$ are then i.i.d. over $(0,1)$ and you are looking for $P(A+Bgefrac43)$
$endgroup$
– Daniel Mathias
Mar 21 at 14:52










2 Answers
2






active

oldest

votes


















3












$begingroup$

Does this make sense to you? I think this aligns with what Ethan has commented.



enter image description here






share|cite|improve this answer









$endgroup$












  • $begingroup$
    Yeah. I got it.
    $endgroup$
    – ChakSayantan
    Mar 21 at 14:47


















0












$begingroup$

A picture, as provided by J. Wang, is very useful for intuition (and has my +1!), but it does need to be made into a rigorous proof. My suggestion is to try conditioning on the first point, $ell_1$. Once you know this, you can ask "Which options for $ell_2$ give the required condition?"---this clearly depends on $ell_1$.



  • If $ell_1 > L/3$, then clearly no $ell_2$ will suffice.

  • If $ell_1 in [0,L/3]$, then any $ell_2 in [ell_1+2L/3,L]$ will suffice.

Given $ell_1$, and that we land in the second scenario, there is a probability $L - (ell_1 + 2L/3) = L/3 - ell_1$ that we have the desired property.
Averaging over $ell_1$, we find that the desired probability is
$$ textstyle int_0^L/3 (L/3 - ell) f_1(ell) dell
quadtextwherequad
f_1(ell) = 1/(L/2) = 2/L text is the pdf of $ell_1$. $$

Note that this integral is equal to
$$ textstyle (2/L) int_0^L/3 ell' d ell' = (2/L) cdot tfrac12 (L/3)^2 = L/9. $$






share|cite|improve this answer









$endgroup$













    Your Answer





    StackExchange.ifUsing("editor", function ()
    return StackExchange.using("mathjaxEditing", function ()
    StackExchange.MarkdownEditor.creationCallbacks.add(function (editor, postfix)
    StackExchange.mathjaxEditing.prepareWmdForMathJax(editor, postfix, [["$", "$"], ["\\(","\\)"]]);
    );
    );
    , "mathjax-editing");

    StackExchange.ready(function()
    var channelOptions =
    tags: "".split(" "),
    id: "69"
    ;
    initTagRenderer("".split(" "), "".split(" "), channelOptions);

    StackExchange.using("externalEditor", function()
    // Have to fire editor after snippets, if snippets enabled
    if (StackExchange.settings.snippets.snippetsEnabled)
    StackExchange.using("snippets", function()
    createEditor();
    );

    else
    createEditor();

    );

    function createEditor()
    StackExchange.prepareEditor(
    heartbeatType: 'answer',
    autoActivateHeartbeat: false,
    convertImagesToLinks: true,
    noModals: true,
    showLowRepImageUploadWarning: true,
    reputationToPostImages: 10,
    bindNavPrevention: true,
    postfix: "",
    imageUploader:
    brandingHtml: "Powered by u003ca class="icon-imgur-white" href="https://imgur.com/"u003eu003c/au003e",
    contentPolicyHtml: "User contributions licensed under u003ca href="https://creativecommons.org/licenses/by-sa/3.0/"u003ecc by-sa 3.0 with attribution requiredu003c/au003e u003ca href="https://stackoverflow.com/legal/content-policy"u003e(content policy)u003c/au003e",
    allowUrls: true
    ,
    noCode: true, onDemand: true,
    discardSelector: ".discard-answer"
    ,immediatelyShowMarkdownHelp:true
    );



    );













    draft saved

    draft discarded


















    StackExchange.ready(
    function ()
    StackExchange.openid.initPostLogin('.new-post-login', 'https%3a%2f%2fmath.stackexchange.com%2fquestions%2f3156880%2fprobability-of-chossing-two-points-from-a-segment-of-length-l-so-that-one-is-2l%23new-answer', 'question_page');

    );

    Post as a guest















    Required, but never shown

























    2 Answers
    2






    active

    oldest

    votes








    2 Answers
    2






    active

    oldest

    votes









    active

    oldest

    votes






    active

    oldest

    votes









    3












    $begingroup$

    Does this make sense to you? I think this aligns with what Ethan has commented.



    enter image description here






    share|cite|improve this answer









    $endgroup$












    • $begingroup$
      Yeah. I got it.
      $endgroup$
      – ChakSayantan
      Mar 21 at 14:47















    3












    $begingroup$

    Does this make sense to you? I think this aligns with what Ethan has commented.



    enter image description here






    share|cite|improve this answer









    $endgroup$












    • $begingroup$
      Yeah. I got it.
      $endgroup$
      – ChakSayantan
      Mar 21 at 14:47













    3












    3








    3





    $begingroup$

    Does this make sense to you? I think this aligns with what Ethan has commented.



    enter image description here






    share|cite|improve this answer









    $endgroup$



    Does this make sense to you? I think this aligns with what Ethan has commented.



    enter image description here







    share|cite|improve this answer












    share|cite|improve this answer



    share|cite|improve this answer










    answered Mar 21 at 14:42









    J. WangJ. Wang

    1916




    1916











    • $begingroup$
      Yeah. I got it.
      $endgroup$
      – ChakSayantan
      Mar 21 at 14:47
















    • $begingroup$
      Yeah. I got it.
      $endgroup$
      – ChakSayantan
      Mar 21 at 14:47















    $begingroup$
    Yeah. I got it.
    $endgroup$
    – ChakSayantan
    Mar 21 at 14:47




    $begingroup$
    Yeah. I got it.
    $endgroup$
    – ChakSayantan
    Mar 21 at 14:47











    0












    $begingroup$

    A picture, as provided by J. Wang, is very useful for intuition (and has my +1!), but it does need to be made into a rigorous proof. My suggestion is to try conditioning on the first point, $ell_1$. Once you know this, you can ask "Which options for $ell_2$ give the required condition?"---this clearly depends on $ell_1$.



    • If $ell_1 > L/3$, then clearly no $ell_2$ will suffice.

    • If $ell_1 in [0,L/3]$, then any $ell_2 in [ell_1+2L/3,L]$ will suffice.

    Given $ell_1$, and that we land in the second scenario, there is a probability $L - (ell_1 + 2L/3) = L/3 - ell_1$ that we have the desired property.
    Averaging over $ell_1$, we find that the desired probability is
    $$ textstyle int_0^L/3 (L/3 - ell) f_1(ell) dell
    quadtextwherequad
    f_1(ell) = 1/(L/2) = 2/L text is the pdf of $ell_1$. $$

    Note that this integral is equal to
    $$ textstyle (2/L) int_0^L/3 ell' d ell' = (2/L) cdot tfrac12 (L/3)^2 = L/9. $$






    share|cite|improve this answer









    $endgroup$

















      0












      $begingroup$

      A picture, as provided by J. Wang, is very useful for intuition (and has my +1!), but it does need to be made into a rigorous proof. My suggestion is to try conditioning on the first point, $ell_1$. Once you know this, you can ask "Which options for $ell_2$ give the required condition?"---this clearly depends on $ell_1$.



      • If $ell_1 > L/3$, then clearly no $ell_2$ will suffice.

      • If $ell_1 in [0,L/3]$, then any $ell_2 in [ell_1+2L/3,L]$ will suffice.

      Given $ell_1$, and that we land in the second scenario, there is a probability $L - (ell_1 + 2L/3) = L/3 - ell_1$ that we have the desired property.
      Averaging over $ell_1$, we find that the desired probability is
      $$ textstyle int_0^L/3 (L/3 - ell) f_1(ell) dell
      quadtextwherequad
      f_1(ell) = 1/(L/2) = 2/L text is the pdf of $ell_1$. $$

      Note that this integral is equal to
      $$ textstyle (2/L) int_0^L/3 ell' d ell' = (2/L) cdot tfrac12 (L/3)^2 = L/9. $$






      share|cite|improve this answer









      $endgroup$















        0












        0








        0





        $begingroup$

        A picture, as provided by J. Wang, is very useful for intuition (and has my +1!), but it does need to be made into a rigorous proof. My suggestion is to try conditioning on the first point, $ell_1$. Once you know this, you can ask "Which options for $ell_2$ give the required condition?"---this clearly depends on $ell_1$.



        • If $ell_1 > L/3$, then clearly no $ell_2$ will suffice.

        • If $ell_1 in [0,L/3]$, then any $ell_2 in [ell_1+2L/3,L]$ will suffice.

        Given $ell_1$, and that we land in the second scenario, there is a probability $L - (ell_1 + 2L/3) = L/3 - ell_1$ that we have the desired property.
        Averaging over $ell_1$, we find that the desired probability is
        $$ textstyle int_0^L/3 (L/3 - ell) f_1(ell) dell
        quadtextwherequad
        f_1(ell) = 1/(L/2) = 2/L text is the pdf of $ell_1$. $$

        Note that this integral is equal to
        $$ textstyle (2/L) int_0^L/3 ell' d ell' = (2/L) cdot tfrac12 (L/3)^2 = L/9. $$






        share|cite|improve this answer









        $endgroup$



        A picture, as provided by J. Wang, is very useful for intuition (and has my +1!), but it does need to be made into a rigorous proof. My suggestion is to try conditioning on the first point, $ell_1$. Once you know this, you can ask "Which options for $ell_2$ give the required condition?"---this clearly depends on $ell_1$.



        • If $ell_1 > L/3$, then clearly no $ell_2$ will suffice.

        • If $ell_1 in [0,L/3]$, then any $ell_2 in [ell_1+2L/3,L]$ will suffice.

        Given $ell_1$, and that we land in the second scenario, there is a probability $L - (ell_1 + 2L/3) = L/3 - ell_1$ that we have the desired property.
        Averaging over $ell_1$, we find that the desired probability is
        $$ textstyle int_0^L/3 (L/3 - ell) f_1(ell) dell
        quadtextwherequad
        f_1(ell) = 1/(L/2) = 2/L text is the pdf of $ell_1$. $$

        Note that this integral is equal to
        $$ textstyle (2/L) int_0^L/3 ell' d ell' = (2/L) cdot tfrac12 (L/3)^2 = L/9. $$







        share|cite|improve this answer












        share|cite|improve this answer



        share|cite|improve this answer










        answered Mar 21 at 22:20









        Sam TSam T

        3,9401031




        3,9401031



























            draft saved

            draft discarded
















































            Thanks for contributing an answer to Mathematics Stack Exchange!


            • Please be sure to answer the question. Provide details and share your research!

            But avoid


            • Asking for help, clarification, or responding to other answers.

            • Making statements based on opinion; back them up with references or personal experience.

            Use MathJax to format equations. MathJax reference.


            To learn more, see our tips on writing great answers.




            draft saved


            draft discarded














            StackExchange.ready(
            function ()
            StackExchange.openid.initPostLogin('.new-post-login', 'https%3a%2f%2fmath.stackexchange.com%2fquestions%2f3156880%2fprobability-of-chossing-two-points-from-a-segment-of-length-l-so-that-one-is-2l%23new-answer', 'question_page');

            );

            Post as a guest















            Required, but never shown





















































            Required, but never shown














            Required, but never shown












            Required, but never shown







            Required, but never shown

































            Required, but never shown














            Required, but never shown












            Required, but never shown







            Required, but never shown







            Popular posts from this blog

            Lowndes Grove History Architecture References Navigation menu32°48′6″N 79°57′58″W / 32.80167°N 79.96611°W / 32.80167; -79.9661132°48′6″N 79°57′58″W / 32.80167°N 79.96611°W / 32.80167; -79.9661178002500"National Register Information System"Historic houses of South Carolina"Lowndes Grove""+32° 48' 6.00", −79° 57' 58.00""Lowndes Grove, Charleston County (260 St. Margaret St., Charleston)""Lowndes Grove"The Charleston ExpositionIt Happened in South Carolina"Lowndes Grove (House), Saint Margaret Street & Sixth Avenue, Charleston, Charleston County, SC(Photographs)"Plantations of the Carolina Low Countrye

            random experiment with two different functions on unit interval Announcing the arrival of Valued Associate #679: Cesar Manara Planned maintenance scheduled April 23, 2019 at 00:00UTC (8:00pm US/Eastern)Random variable and probability space notionsRandom Walk with EdgesFinding functions where the increase over a random interval is Poisson distributedNumber of days until dayCan an observed event in fact be of zero probability?Unit random processmodels of coins and uniform distributionHow to get the number of successes given $n$ trials , probability $P$ and a random variable $X$Absorbing Markov chain in a computer. Is “almost every” turned into always convergence in computer executions?Stopped random walk is not uniformly integrable

            How should I support this large drywall patch? Planned maintenance scheduled April 23, 2019 at 00:00UTC (8:00pm US/Eastern) Announcing the arrival of Valued Associate #679: Cesar Manara Unicorn Meta Zoo #1: Why another podcast?How do I cover large gaps in drywall?How do I keep drywall around a patch from crumbling?Can I glue a second layer of drywall?How to patch long strip on drywall?Large drywall patch: how to avoid bulging seams?Drywall Mesh Patch vs. Bulge? To remove or not to remove?How to fix this drywall job?Prep drywall before backsplashWhat's the best way to fix this horrible drywall patch job?Drywall patching using 3M Patch Plus Primer